next up previous
Next: Solution to Problem 2. Up: No Title Previous: No Title

Solution to Problem 1.

1. [30 points] Let tex2html_wrap_inline598 , tex2html_wrap_inline600 , tex2html_wrap_inline602 be independent discrete random variables with

  equation138

Here, the tex2html_wrap_inline604 , tex2html_wrap_inline606 , tex2html_wrap_inline602 is a sequence of positive constants and tex2html_wrap_inline610 , tex2html_wrap_inline612 , tex2html_wrap_inline602 is a sequence of constants satisfying tex2html_wrap_inline616 for all j.

Define

   eqnarray147

Assume

  equation154

Show that tex2html_wrap_inline620 .

Solution: We will use the following version of the Lindeberg C.L.T. (this is the one stated in class):

Let tex2html_wrap_inline622 be a triangular array of random variables satisfying

(i)
tex2html_wrap_inline624 as tex2html_wrap_inline626 ;
(ii)
(Rowwise independence) For each n, the random variables tex2html_wrap_inline630 , tex2html_wrap_inline632 , tex2html_wrap_inline602 , tex2html_wrap_inline636 are mutually independent.
(iii)
(Centered) tex2html_wrap_inline638 = 0 for all n and k.
(iv)
(Normalized) tex2html_wrap_inline648 = 1.
(v)
(Lindeberg Condition) For every tex2html_wrap_inline654 ,

displaymath586

Then

displaymath587

Now

  equation182

so we don't need to do any centering. Also,

displaymath588

Therefore,

  equation185

where tex2html_wrap_inline656 is given in (3). Thus, if we define

displaymath589

then we claim the conditions of the Lindeberg CLT hold. We have tex2html_wrap_inline658 so condition (i) holds. Condition (ii) is immediate since we are given that the tex2html_wrap_inline660 's are independent. Condition (iii) follows from (5). For condition (iv), observe that

displaymath590

by (6). Finally, to verify (v), let tex2html_wrap_inline654 be given. Then

   eqnarray196

Of course, tex2html_wrap_inline664 is 0 or tex2html_wrap_inline668 , so tex2html_wrap_inline670 if and only if tex2html_wrap_inline672 . But by (4) there exists an tex2html_wrap_inline674 such that for all tex2html_wrap_inline676 ,

  equation208

Hence, for all tex2html_wrap_inline676 ,

  equation212

and so for all tex2html_wrap_inline676 ,

  equation215

This proves the Lindeberg condition holds. Hence, we conclude

displaymath591

as required.



Dennis Cox
Tue Nov 11 20:55:50 CST 1997